1.2

Pataasin ang iyong marka sa homework at exams ngayon gamit ang Quizwiz!

BE&H Manufacturing is considering dropping a product line. It currently produces a multi-purpose woodworking clamp in a simple manufacturing process that uses special equipment. Variable costs amount to $6.00 per unit. Fixed overhead costs, exclusive of depreciation, have been allocated to this product at a rate of $3.50 a unit and will continue whether or not production ceases. Depreciation on the special equipment amounts to $20,000 a year. If production of the clamp is stopped, the special equipment can be sold for $18,000; if production continues, however, the equipment will be useless for further production at the end of 1 year and will have no salvage value. The clamp has a selling price of $10 a unit. Ignoring tax effects, the minimum number of units that would have to be sold in the current year to break even on a cash flow basis is A. 4,500 units. B. 5,000 units. C. 20,000 units. D. 36,000 units.

Answer (A) is correct. The BEP in units is equal to fixed costs divided by the difference between unit selling price and unit variable cost (UCM). The $18,000 salvage value, the cash flow to be received if production is discontinued, is treated here as a fixed cost. Hence, continuation of the product line will permit the firm to break even or make a profit only if the total CM is $18,000 or more.

A company has sales of $500,000, variable costs of $300,000, and pretax profit of $150,000. If the company increased the sales price per unit by 10%, reduced fixed costs by 20%, and left variable cost per unit unchanged, what would be the new breakeven point in sales dollars? A. $88,000 B. $100,000 C. $110,000 D. $125,000

Answer (A) is correct. The breakeven point in sales dollars is equal to the sum of fixed costs plus any desired pretax profit, divided by the contribution margin ratio [(sales - variable costs) ÷ sales]. Fixed cost was $50,000 ($500,000 sales - $300,000 VC - $150,000 pretax profit). Given the increase in sales of 10% and decrease in fixed costs of 20%, the breakeven point in sales is $88,000. ($50,000 × .8) Fixed Costs + $0 Desired Pretax Profit [($500,000 × 1.1) Sales - Variable Costs ($300,000)] ÷ Sales ($500,000 × 1.1) = $40,000 = $250,000 ÷ $550,000

The breakeven point in units increases when unit costs A. Increase and sales price remains unchanged. B. Decrease and sales price remains unchanged. C. Remain unchanged and sales price increases. D. Decrease and sales price increases.

Answer (A) is correct. The breakeven point in units is calculated by dividing the fixed costs by the contribution margin per unit. If selling price is constant and costs increase, the unit contribution margin will decline, resulting in an increase of the breakeven point. Answer (B) is incorrect because A decrease in costs will lower the breakeven point. The unit contribution margin will increase. Answer (C) is incorrect because An increase in the selling price will also increase the unit contribution margin, resulting in a lower breakeven point. Answer (D) is incorrect because Both a cost decrease and a sales price increase will increase the unit contribution margin, resulting in a lower breakeven point.

(Refers to Fact Pattern #2) Assuming that XYZ Company sells 80,000 units, what is the maximum that can be paid for an advertising campaign while still breaking even? A. $135,000 B. $1,015,000 C. $535,000 D. $695,000

Answer (A) is correct. The company will break even when net income equals zero. Net income is equal to revenue minus variable expenses and fixed expenses, including advertising. Thus, if X equals advertising cost, the equation is (80,000)($25) - (80,000)($16) - $585,000 - X = 0 $2,000,000 - $1,280,000 - $585,000 - X = 0 X = $135,000

(Refers to Fact Pattern #1) The annual sales volume required for Madengrad Company to break even is A. 22,000 units. B. 11,000 units. C. 8,400 units. D. 13,888 units.

Answer (A) is correct. The formula for the breakeven point in units divides the fixed costs by the unit contribution margin ($900 selling price - $600 variable costs = $300). Hence, the breakeven point is 22,000 units ($6,600,000 ÷ 300). Answer (B) is incorrect because The breakeven point is determined by dividing the fixed costs ($6,600,000) by the unit contribution margin ($900 selling price - $600 variable costs = $300). Answer (C) is incorrect because The breakeven point is determined by dividing the fixed costs ($6,600,000) by the unit contribution margin ($900 selling price - $600 variable costs = $300). Answer (D) is incorrect because The breakeven point is determined by dividing the fixed costs ($6,600,000) by the unit contribution margin ($900 selling price - $600 variable costs = $300).

(Refers to Fact Pattern #4) The maximum after-tax profit that can be earned by Delphi Company from sales of the new product during the next fiscal year is A. $30,000 B. $50,000 C. $110,000 D. $66,000

Answer (A) is correct. The maximum output (maximum sales level) is 25,000 units (given). Thus, the breakeven point in units is 22,500 ($450,000FC ÷ $20). The unit contribution margin is $20 ($36 selling price - $16 unit VC). At the breakeven point, all fixed costs have been recovered. Hence, pretax profit equals the unit contribution margin times unit sales in excess of the breakeven point, or $50,000 [(25,000 unit sales - 22,500 BEP) × $20 UCM]. After-tax profit is $30,000 [$50,000 × (1.0 - .4 tax rate)].

Which of the following would decrease unit contribution margin the most? A. A 15% decrease in selling price. B. A 15% increase in variable expenses. C. A 15% decrease in variable expenses. D. A 15% decrease in fixed expenses.

Answer (A) is correct. The plausible options are a decrease in selling price or an increase in variable expenses because UCM equals unit sales price minus unit variable cost. However, a given percentage change in unit sales price must have a greater effect than an equal but opposite percentage change in unit variable cost because the former is greater than the latter. The example below demonstrates this point. Original: SP = VC = CM = SP - 15%: SP = VC = CM = VC + 15%: SP = VC = CM = $100 50 $ 50 $ 85 50 $ 35 ($15 decrease) $100.00 57.50 $ 42.50 ($7.50 decrease) Answer (B) is incorrect because A 15% increase in variable expenses will not decrease the CM as much as a 15% decrease in sales price. Answer (C) is incorrect because A decrease in variable expenses would increase UCM. Answer (D) is incorrect because A decrease in fixed expenses has no effect on the UCM.

(Refers to Fact Pattern #5) If the selling price is $14 per unit, the breakeven point in units (rounded to the nearest hundred) for surge protectors is A. 8,500 units. B. 10,000 units. C. 15,000 units. D. 20,000 units.

Answer (A) is incorrect because A breakeven point of 8,500 units ignores variable costs. Answer (B) is incorrect because The breakeven point is 20,000 units when the contribution margin is $6 per unit. Answer (C) is incorrect because This number of units equals fixed costs divided by unit variable cost. Answer (D) is correct. The breakeven point in units equals total additional fixed costs divided by the unit contribution margin. Unit variable costs total $8 ($3.25 + $4.00 + $.75). Thus, UCM is $6 ($14 unit selling price - $6 unit VC), and the breakeven point is 20,000 units ($120,000 FC ÷ $6).

One of the major assumptions limiting the reliability of breakeven analysis is that A. Efficiency and productivity will continually increase. B. Total variable costs will remain unchanged over the relevant range. C. Total fixed costs will remain unchanged over the relevant range. D. The cost of production factors varies with changes in technology

Answer (A) is incorrect because Breakeven analysis assumes no changes in efficiency and productivity. Answer (B) is incorrect because One limiting assumption is that unit variable cost, not total variable cost, is constant. Answer (C) is correct. The inherent simplifying assumptions used in CVP analysis are the following: Costs and revenues are predictable and are linear over the relevant range; variable costs change proportionally with activity level; changes in inventory are insignificant in amount; fixed costs remain constant over the relevant range of volume; prices remain fixed; production equals sales; there is a relevant range in which the various relationships are true for a given time span; all costs are either fixed or variable; productive efficiency is constant; costs vary only with changes in sales volume; and there is a constant mix of products (or only one product). Answer (D) is incorrect because The cost of production factors is assumed to be stable.

(Refers to Fact Pattern #1) If Madengrad Company achieves a sales and production volume of 8,000 units, the annual before-tax income (loss) will be A. $(4,200,000) B. $1,780,000 C. $(2,520,000) D. $(420,000)

Answer (A) is correct. At a volume of 8,000 units, sales will be $7,200,000 (8,000 units × $900). Variable costs will be $4,800,000 (8,000 units × $600). Thus, the contribution margin is $2,400,000. Deducting the $6,600,000 of fixed costs from the contribution margin leaves a net loss of $4,200,000. Answer (B) is incorrect because The annual before-tax (loss) is $(4,200,000) ($7,200,000 sales - $4,800,000 variable costs - $6,600,000 fixed costs). Answer (C) is incorrect because The annual before-tax (loss) is $(4,200,000) ($7,200,000 sales - $4,800,000 variable costs - $6,600,000 fixed costs). Answer (D) is incorrect because The annual before-tax (loss) is $(4,200,000) ($7,200,000 sales - $4,800,000 variable costs - $6,600,000 fixed costs).

Two companies produce and sell the same product in a competitive industry. Thus, the selling price of the product for each company is the same. Company 1 has a contribution margin ratio of 40% and fixed costs of $25 million. Company 2 is more automated, making its fixed costs 40% higher than those of Company 1. Company 2 also has a contribution margin ratio that is 30% greater than that of Company 1. By comparison, Company 1 will have the <List A> breakeven point in terms of dollar sales volume and will have the <List B> dollar profit potential once the indifference point in dollar sales volume is exceeded. List A A. Lower B. Lower C. Higher D. Higher List B Lesser Greater Lesser Greater

Answer (A) is correct. Company 1's breakeven point is lower because its fixed costs are lower. Company 1's breakeven point is $62,500,000 ($25,000,000 ÷ 40%). Company 2's breakeven point is $67,307,692 [($25,000,000 × 1.4) ÷ (40% × 1.3)]. The indifference point, at which dollar profits are equal, is $83,333,333 ($25,000,000 + .60X = $35,000,000 + .48X). Once the indifference point is passed, Company 1 will make lower profits than Company 2 because Company 2 has a higher contribution margin. Answer (B) is incorrect because Company 1's breakeven point is lower, but its profits will be lower past the indifference point. Answer (C) is incorrect because Company 1's breakeven point is lower, but its profits will be lower past the indifference point. Answer (D) is incorrect because Company 1's breakeven point is lower, but its profits will be lower past the indifference point.

(Refers to Fact Pattern #3) What is the average cost per unit at a production level of 8,000 units for cost X? A. $5.98 B. $5.85 C. $7.90 D. $4.83

Answer (A) is correct. Cost X is a mixed cost (part variable and part fixed). The hi-low method can be used to determine the fixed and variable portions. Dividing the total cost of X at two different production volumes by the difference in units produced gives a unit variable cost of $4.83 [($178,260 - $23,700) ÷ (35,000 units - 3,000 units)]. Fixed cost must therefore be $9,210 [$178,260 cost of X for 35,000 units - (35,000 units × $4.83)]. Total cost of X for 8,000 units is $47,850 [$9,210 + (8,000 units × $4.83)], and the average cost is $5.98 ($47,850 ÷ 8,000).

How much does each additional sales dollar contribute toward profit for a firm with $4 million break-even level of revenues and $1.2 million in fixed costs including depreciation? A. $0.30 B. $0.33 C. $0.50 D. $0.67

Answer (A) is correct. The $4 million breakeven level was calculated as follows: Solving for CM% results in CM% = 30%. Thus, 30% of every dollar, or $0.30, contributes towards profits. Answer (B) is incorrect because When substituted into the original breakeven formula, it would not have produced a breakeven level of $4 million. Answer (C) is incorrect because When substituted into the original breakeven formula, it would not have produced a breakeven level of $4 million. Answer (D) is incorrect because When substituted into the original breakeven formula, it would not have produced a breakeven level of $4 million.

When used in cost-volume-profit analysis, sensitivity analysis A. Determines the most profitable mix of products to be sold. B. Allows the decision maker to introduce probabilities in the evaluation of decision alternatives. C. Is done through various possible scenarios and computes the impact on profit of various predictions of future events. D. Is limited because, in cost-volume-profit analysis, costs are not separated into fixed and variable components.

Answer (A) is incorrect because CVP analysis assumes a constant product mix or only one product. Answer (B) is incorrect because Expected value analysis allows the decision maker to introduce probabilities in the evaluation of decision alternatives. Answer (C) is correct. Sensitivity analysis permits the decision maker to measure the effects of errors in certainty equivalents, which are estimated amounts developed by the best means available and assumed for purposes of a given decision model to be certain. The decision model then may be evaluated by changing certain data variables (certainty equivalents) critical to the success of the entity and observing the outcomes. This analysis allows the decision maker to quantify the effects of forecasting or prediction errors and to identify the most critical variables. For example, with respect to breakeven analysis, a firm might make computations using several different estimates of what fixed costs are expected to be. These calculations indicate how sensitive the results are to changes in fixed costs. Answer (D) is incorrect because In cost-volume-profit analysis, costs are separated into fixed and variable components.

Which of the following will result in raising the breakeven point? A. A decrease in the variable cost per unit. B. An increase in the semivariable cost per unit. C. An increase in the contribution margin per unit. D. A decrease in income tax rates.

Answer (A) is incorrect because If other factors are constant, an increase in sales price or a decrease in unit variable cost increases the CM and lowers the BEP. Answer (B) is correct. The BEP equals fixed cost divided by the UCM (selling price - unit variable cost). An increase in semivariable costs increases fixed costs and/or variable costs. An increase in either will raise the BEP. If fixed costs increase, more units must be sold, assuming the same UCM, to cover the greater fixed costs. If variable costs increase, the UCM will decrease and again more units must be sold to cover the fixed costs. Answer (C) is incorrect because An increase in the CM decreases the BEP. Answer (D) is incorrect because If income taxes are taken into account, they are treated as variable costs. A decrease in variable costs lowers the BEP.

In working on a CVP analysis, the accountant is unsure of the exact results and/or assumptions under which to operate. What can the accountant do to help management in this CVP decision? A. Nothing. It is not the responsibility of the accountant to be concerned with the ambiguity of the results and/or assumptions. B. Ascertain the probabilities of various outcomes and work with management on understanding those probabilities in reference to the CVP decision. C. Calculate the probabilities of various outcomes and make the decision for management. D. Use a random number table to generate a decision model and make the decision for management.

Answer (A) is incorrect because It is the responsibility of the accountant to be involved with management and aid in their decision making. Answer (B) is correct. The assumptions under which CVP analysis operates primarily hinge on certainty. Once uncertainty enters the situation, the results are not so clear. Thus, the accountant should make an appropriate effort to ascertain the probabilities of various outcomes. The accountant can then work with management to help make the appropriate decision. Answer (C) is incorrect because It is not appropriate for the accountant to make the decision. Management should make the decision with the accountant's advice and help. Answer (D) is incorrect because Although using a random number table or a simulation may be part of the decision process, the decision should not rest with the accountant alone.

A manufacturer contemplates a change in technology that would reduce fixed costs from $800,000 to $700,000. However, the ratio of variable costs to sales will increase from 68% to 80%. What will happen to break-even level of revenues? A. Decrease by $301,470.50. B. Decrease by $500,000. C. Decrease by $1,812,500. D. Increase by $1,000,000.

Answer (A) is incorrect because It uses the variable cost percentage in the denominator instead of the contribution margin percentage. Answer (B) is incorrect because It uses the same contribution margin percentage (20%) in both calculations. Answer (C) is incorrect because It reverses the two contribution margin percentages. Answer (D) is correct. The original breakeven level was: The new level is: Thus, there is an increase of $1,000,000.

If inventories are expected to change, the type of costing that provides the best information for breakeven analysis is A. Job order costing. B. Variable (direct) costing. C. Joint costing. D. Absorption (full) costing.

Answer (A) is incorrect because Job order costing does not separate fixed costs from variable costs. Answer (B) is correct. A variable (direct) costing system is best for providing the information needed for CVP analysis because both techniques separate the fixed costs from variable costs. CVP analysis calculates a variable cost per unit and deducts it from unit sales price to determine the unit contribution margin. The total contribution margin from a given level of unit sales measures the extent of recovery of fixed costs and the profit earned. Direct costing is likewise oriented toward determination of the contribution margin because it treats fixed manufacturing overhead as a period, not a product, cost. Thus, it facilitates CVP analysis by isolating variable manufacturing costs. Answer (C) is incorrect because Joint costing does not separate fixed costs from variable costs. Answer (D) is incorrect because Absorption (full) costing does not separate fixed costs from variable costs.

Marston Enterprises sells three chemicals: petrol, septine, and tridol. Petrol is the company's most profitable product; tridol is the least profitable. Which one of the following events will definitely decrease the firm's overall breakeven point for the upcoming accounting period? A. The installation of new computer-controlled machinery and subsequent layoff of assembly-line workers. B. A decrease in tridol's selling price. C. An increase in the overall market for septine. D. An increase in anticipated sales of petrol relative to sales of septine and tridol.

Answer (A) is incorrect because The acquisition of new machinery will result in greater fixed costs and the possibility of a higher breakeven point. Answer (B) is incorrect because A decrease in selling price reduces the unit contribution margin, which in turn increases the breakeven point. Answer (C) is incorrect because An increase in the market for septine has an indeterminate effect. The facts given do not indicate whether its unit contribution margin is greater or less than the average unit contribution margin for all products. Answer (D) is correct. A company's breakeven point will be reduced if fixed costs are lowered or the average unit contribution margin is increased. Given that petrol is the company's most profitable product, and assuming that it has a higher unit contribution margin than septine and tridol, an increase in sales of petrol relative to the other products will result in a higher average unit contribution margin and a lower breakeven point (fixed costs ÷ average UCM).

(Refers to Fact Pattern #1) For Madengrad Company to achieve an after-tax net income of $540,000, annual sales revenue must be A. $23,850,000 B. $22,500,000 C. $2,700,000 D. $21,420,000

Answer (A) is incorrect because The breakeven point in sales dollars is $22,500,000 [($6,600,000 fixed costs + $900,000 before-tax income) ÷ 33 1/3% contribution margin percentage]. Answer (B) is correct. The formula for the breakeven point in sales divides fixed costs by the contribution margin percentage. The contribution margin percentage is 33 1/3% [$300 unit contribution margin ($900 selling price - $600 variable costs) ÷ $900 selling price]. The after-tax income specified is $540,000, which is equal to 60% (1.0 - .4 tax rate) of the before-tax income. Consequently, the before-tax income needed is $900,000 ($540,000 ÷ .6). This amount may be treated as a fixed cost for the purpose of the formula. The breakeven point in sales dollar is therefore $22,500,000 [($6,600,000 + $900,000) ÷ 33 1/3%]. Answer (C) is incorrect because The breakeven point in sales dollars is $22,500,000 [($6,600,000 fixed costs + $900,000 before-tax income) ÷ 33 1/3% contribution margin percentage]. Answer (D) is incorrect because The breakeven point in sales dollars is $22,500,000 [($6,600,000 fixed costs + $900,000 before-tax income) ÷ 33 1/3% contribution margin percentage].

A company's breakeven point in sales dollars may be affected by equal percentage increases in both selling price and variable cost per unit (assume all other factors are constant within the relevant range). The equal percentage changes in selling price and variable cost per unit will cause the breakeven point in sales dollars to A. Decrease by less than the percentage increase in selling price. B. Decrease by more than the percentage increase in the selling price. C. Increase by the percentage change in variable cost per unit. D. Remain unchanged.

Answer (A) is incorrect because The breakeven point in sales dollars will not change. Answer (B) is incorrect because The CMR will remain the same. Therefore, the breakeven point in sales dollars will remain unchanged. Answer (C) is incorrect because The breakeven point in sales dollars will not change. Answer (D) is correct. The BEP in sales dollars is equal to the fixed cost divided by the CMR. Accordingly, equal percentage changes in selling price and variable cost per unit will not affect the BEP in sales dollars. For example, assume the unit price of a product is $1 and its unit variable cost is $.60. The CMR equals 40% [($1 - $.60)UCM ÷ $1 unit price]. If fixed cost is $100, the BEP in sales dollars is $250 ($100 ÷ 40%). Raising the selling price and variable cost by 20% to $1.20 and $.72, respectively, leaves the CMR at 40% ($.48 ÷ $1.20). Similarly, lowering the selling price and variable cost to $.80 and $.48, respectively, also leaves the CMR at 40% ($.32 ÷ $.80).

Which of the following is a characteristic of a contribution income statement? A. Fixed and variable expenses are combined as one line. B. Fixed expenses are listed separately from variable expenses. C. Fixed and variable manufacturing costs are combined as one line item, but fixed operating expenses are shown separately from variable operating expenses. D. Fixed and variable operating expenses are combined as one line item, but fixed manufacturing expenses are shown separately from variable manufacturing expenses.

Answer (A) is incorrect because The contribution income statement shows the contribution margin (sales - variable costs) before it subtracts the fixed costs. Fixed costs are not combined with variable costs. Answer (B) is correct. The contribution income statement emphasizes the distinction between fixed and variable costs. Making this distinction facilitates determination of CVP relationships and the effects of changes in sales volume on income. Thus, fixed manufacturing costs and other fixed costs are separated from variable manufacturing costs and other variable costs. The basic categories in the contribution income statement are variable costs, contribution margin, fixed costs, and operating income. Answer (C) is incorrect because Fixed costs are not combined with variable costs on contribution income statements. Answer (D) is incorrect because Fixed costs are not combined with variable costs on contribution income statements.

The margin of safety is a key concept of CVP analysis. The margin of safety is the A. Contribution margin rate. B. Difference between budgeted contribution margin and breakeven contribution margin. C. Difference between budgeted sales and breakeven sales.

Answer (A) is incorrect because The contribution margin rate is computed by dividing contribution margin by sales. The contribution margin equals sales minus total variable costs. Answer (B) is incorrect because The margin of safety is expressed in revenue or units, not contribution margin. Answer (C) is correct. The margin of safety measures the amount by which sales may decline before losses occur. It is the excess of budgeted or actual sales over sales at the BEP. It may be stated in either units sold or sales revenue. Answer (D) is incorrect because Cash flow is not relevant.

Cost-volume-profit (CVP) analysis is a key factor in many decisions, including choice of product lines, pricing of products, marketing strategy, and use of productive facilities. A calculation used in a CVP analysis is the breakeven point. Once the breakeven point has been reached, operating income will increase by the A. Gross margin per unit for each additional unit sold. B. Contribution margin per unit for each additional unit sold. C. Fixed costs per unit for each additional unit sold. D. Variable costs per unit for each additional unit sold.

Answer (A) is incorrect because The gross margin equals sales price minus cost of goods sold, including fixed cost. Answer (B) is correct. At the breakeven point, total revenue equals the fixed cost plus the variable cost. Beyond the BEP, each unit sale will increase operating income by the unit contribution margin (unit sales price - unit variable cost) because fixed cost will already have been recovered. Answer (C) is incorrect because Operating income will increase by the UCM. Answer (D) is incorrect because Operating income will increase by the UCM.

(Refers to Fact Pattern #1) Assume a 10% increase in annual fixed costs, a 20% unit cost increase for direct labor, and a reduction in unit material costs of 25%, with no change in selling price. Madengrad Company's breakeven point would increase (decrease) (rounded to the nearest whole unit) by A. 3,960 units. B. (1,620) units. C. 1,604 units. D. 407 units.

Answer (A) is incorrect because The new breakeven point is 22,407 units [$7,260,000 ($6,600,000 × 1.1) fixed costs ÷ $324]. The new breakeven point is an increase of 407 units (22,407 units - 22,000 units at the previous breakeven point. Answer (B) is incorrect because The new breakeven point is 22,407 units [$7,260,000 ($6,600,000 × 1.1) fixed costs ÷ $324]. The new breakeven point is an increase of 407 units (22,407 units - 22,000 units at the previous breakeven point. Answer (C) is incorrect because The new breakeven point is 22,407 units [$7,260,000 ($6,600,000 × 1.1) fixed costs ÷ $324]. The new breakeven point is an increase of 407 units (22,407 units - 22,000 units at the previous breakeven point. Answer (D) is correct. The new contribution margin is $324. Dividing this amount into the fixed costs will determine the new unit breakeven point. Fixed costs have increased by 10% to $7,260,000 ($6,600,000 × 1.1), and the new breakeven point is 22,407 units ($7,260,000 ÷ $324). The original unit breakeven point was 22,000 units. Hence, the production changes increased the breakeven point by 407 units (22,407 - 22,000).

For a profitable company, the amount by which sales can decline before losses occur is known as the A. Sales volume variance. B. Hurdle rate. C. Variable sales ratio. D. Margin of safety.

Answer (A) is incorrect because The sales quantity (volume) variance focuses on the firm's aggregate results. It assumes a constant product mix and an average contribution margin for the composite unit. The sales volume variance equals the budgeted average UCM calculated for the composite unit multiplied by the difference between the actual and budgeted unit sales. Answer (B) is incorrect because It is the rate of return a potential investment must earn before it is acceptable to management. Answer (C) is incorrect because It is a nonsense term. Answer (D) is correct. The margin of safety measures the amount by which sales may decline before losses occur. It equals budgeted or actual sales minus sales at the BEP. It may be stated in either units sold or sales revenue.

(Refers to Fact Pattern #1) Assume a 10% increase in annual fixed costs, a 20% unit cost increase for direct labor, and a reduction in unit material costs of 25%, with no change in selling price. After incorporating these changes, Madengrad Company's contribution margin would be A. 34% B. 69% C. 36% D. 64%

Answer (A) is incorrect because The unit variable cost is reduced by $24, so UCM is $324. The contribution margin percentage will be 36% ($324 ÷ $900). Answer (B) is incorrect because The unit variable cost is reduced by $24, so UCM is $324. The contribution margin percentage will be 36% ($324 ÷ $900). Answer (C) is correct. The original unit contribution margin (UCM) was $300. But unit direct labor cost is 20% higher ($180 × 20% = $36). Unit direct materials cost will be reduced by 25%, or $60 ($240 × 25%). Thus, the net result is that unit variable cost is reduced by $24 ($60 - $36), and the UCM will be $324 ($300 + $24). The contribution margin percentage will therefore be 36% ($324 ÷ 900). Answer (D) is incorrect because The unit variable cost is reduced by $24, so UCM is $324. The contribution margin percentage will be 36% ($324 ÷ $900).

Product A accounts for 75% of a company's total sales revenue and has a variable cost equal to 60% of its selling price. Product B accounts for 25% of total sales revenue and has a variable cost equal to 85% of its selling price. What is the breakeven point given fixed costs of $150,000? A. $375,000 B. $444,444 C. $500,000 D. $545,455

Answer (A) is incorrect because This amount is based on the contribution margin of Product A only rather than a weighted average. Answer (B) is correct. Sales = VC + FC. The proportion of each product's sales must be considered. S = 0.75S(0.60) + 0.25S(0.85) + $150,000 S = 0.45S + 0.2125S + $150,000 S - 0.6625S = $150,000 0.3375S = $150,000 S = $444,444 Answer (C) is incorrect because This amount is based on half of the required sales at B's contribution margin. Answer (D) is incorrect because This amount is based on an unweighted average of the two contribution margins.

The breakeven point in units sold for Tierson Corporation is 44,000. If fixed costs for Tierson are equal to $880,000 annually and variable costs are $10 per unit, what is the contribution margin per unit for Tierson Corporation? A. $0.05 B. $20.00 C. $44.00 D. $88.00

Answer (A) is incorrect because This amount results from inverting the numerator and denominator in the calculation. Answer (B) is correct. The breakeven point in units is equal to the fixed costs divided by the contribution margin per unit. Thus, the UCM is $20.00 ($880,000 ÷ 44,000 units). Answer (C) is incorrect because This amount results from using variable cost as part of the calculation. Answer (D) is incorrect because This amount results from dividing by an erroneous denominator.

The change in period-to-period operating income when using variable costing can be explained by the change in the A. Unit sales level multiplied by the unit sales price. B. Finished goods inventory level multiplied by the unit sales price. C. Unit sales level multiplied by a constant unit contribution margin. D. Finished goods inventory level multiplied by a constant unit contribution margin

Answer (A) is incorrect because Unit sales multiplied by the sales price equals sales revenue, but this amount does not necessarily correlate with operating income. A change in unit variable costs may cause revenue and operating income to move in different directions. Answer (B) is incorrect because Operating income is not necessarily correlated positively or negatively with finished goods inventory, however valued. Answer (C) is correct. In a variable costing system, only the variable costs are recorded as product costs. All fixed costs are expensed in the period incurred. Because changes in the relationship between production levels and sales levels do not cause changes in the amount of fixed manufacturing cost expensed, profits more directly follow the trends in sales, especially when the UCM (selling price per unit - variable costs per unit) is constant. Unit sales times the UCM equals the total CM, and operating income (a pretax amount) equals the CM minus fixed costs of operations. If the UCM is constant and fixed costs are stable, the change in operating income will approximate the change in the CM (UCM × unit sales). Answer (D) is incorrect because Operating income is not necessarily correlated positively or negatively with finished goods inventory, however valued.

What is the breakeven point in units for a product that sells for $10 if fixed costs are $4,000 and variable costs are 20%? A. 250 B. 500 C. 800 D. 2,000

Answer (A) is incorrect because Using an erroneous contribution margin results in 250. Answer (B) is correct. The breakeven point is where profit is zero and sales = fixed costs + variable costs, so 10x = 4,000 + 2x. Thus, 8x = 4,000, or x = 500 units. Alternatively, dividing the $4,000 of fixed costs by the $8 per unit contribution margin gives the same result. Answer (C) is incorrect because Using the inverse of the contribution margin results in 800. Answer (D) is incorrect because Applying the 20% to fixed costs instead of sales results in 2,000.

Which one of the following is true regarding a relevant range? A. Total variable costs will not change. B. Total fixed costs will not change. C. Actual fixed costs usually fall outside the relevant range. D. The relevant range cannot be changed after being established.

Answer (A) is incorrect because Variable costs will change in total, but unit variable costs will be constant. Answer (B) is correct. The relevant range is the range of activity over which unit variable costs and total fixed costs are constant. The incremental cost of one additional unit of production will be equal to the variable cost. Answer (C) is incorrect because Actual fixed costs should not vary greatly from budgeted fixed costs for the relevant range. Answer (D) is incorrect because The relevant range can change whenever production activity changes; the relevant range is merely an assumption used for budgeting and control purposes.

Associated Supply, Inc. is considering introducing a new product that will require a $250,000 investment of capital. The necessary funds would be raised through a bank loan at an interest rate of 8%. The fixed operating costs associated with the product would be $122,500, while the contribution margin percentage would be 42%. Assuming a selling price of $15 per unit, determine the number of units (rounded to the nearest whole unit) Associated would have to sell to generate earnings before interest and taxes (EBIT) of 32% of the amount of capital invested in the new product. A. 35,318 units. B. 32,143 units. C. 25,575 units. D. 23,276 units

Answer (B) is correct. Associated has determined it must generate EBIT equal to 32% of the capital invested in this project, or $80,000 ($250,000 × 32%). The number of units it must produce to achieve this level of EBIT can be derived as follows: Breakeven point = (EBIT + Fixed costs) ÷ Unit contribution margin = ($80,000 + $122,500) ÷ ($15 × 42%) = $202,500 ÷ $6.30 = 32,142.86 units

A company with $280,000 of fixed costs has the following data: Sales price per unit Variable costs per unit Product A $5 $3 Product B $6 $5 Assume three units of A are sold for each unit of B sold. How much will sales be in dollars of product B at the breakeven point? A. $200,000 B. $240,000 C. $280,000 D. $840,000

Answer (B) is correct. The breakeven point equals fixed costs divided by unit contribution margin. The composite unit contribution margin for A and B is $7 {[3 units of A × ($5 - $3)] + [1 unit of B × ($6 - $5)]}. Thus, 40,000 composite units ($280,000 ÷ $7), including 40,000 units of B, are sold at the breakeven point. Hence, sales of B at the breakeven point equal $240,000 (40,000 units × $6).

A retail company determines its selling price by marking up variable costs 60%. In addition, the company uses frequent selling price markdowns to stimulate sales. If the markdowns average 10%, what is the company's contribution margin ratio? A. 27.5% B. 30.6% C. 37.5% D. 41.7%

Answer (B) is correct. The contribution margin equals revenues minus variable costs. The contribution margin ratio equals the unit contribution margin divided by the selling price. For example, if variable costs average $10 per unit, the average selling price is $16 (1.60 × $10). However, the 10% markdown implies that the actual average selling price is $14.40 (.90 × $16). The contribution margin ratio is therefore 30.6% [($14.40 - $10.00) ÷ $14.40].

(Refers to Fact Pattern #2) How many units does XYZ Company need to produce and sell to make a before-tax profit of 10% of sales? A. 65,000 units. B. 36,562 units. C. 90,000 units. D. 25,000 units.

Answer (C) is correct. Revenue minus variable and fixed expenses equals net income. If X equals unit sales, revenue equals $25X, total variable expenses equal $16X ($4 + $7 + $2 + $3), total fixed expenses equal $585,000 ($360,000 + $225,000), and net income equals 10% of revenue. Hence, X equals 90,000 units. $25X - $16X - $585,000 = $25X × 10% 6.5X = $585,000 X = 90,000 units

(Refers to Fact Pattern #4) The number of units of the new product that Delphi Company must sell during the next fiscal year in order to break even is A. 20,930 B. 18,140 C. 22,500 D. 25,500

Answer (C) is correct. The breakeven point in units equals fixed costs divided by the unit contribution margin (in dollars). The unit contribution margin is $20 ($36 selling price - $16 unit variable cost). Hence, the breakeven point equals 22,500 units ($450,000 ÷ $20).

Positive operating income is shown on a cost-volume-profit chart when the A. Total variable expense line exceeds the total fixed expense line. B. Total expense line exceeds the total sales revenue line. C. Total sales revenue line exceeds the total fixed expense line. D. Total sales revenue line exceeds the total expense line.

Answer (D) is correct. A cost-volume-profit chart contains elements (lines, points, axes) that identify variable cost, fixed cost, the breakeven point, total revenue, profit, and volume in units. When the total sales revenue line rises above the total expense line, a company will have positive net income.

(Refers to Fact Pattern #4) Delphi Company's management has stipulated that it will not approve the continued manufacture of the new product after the next fiscal year unless the after-tax profit is at least $75,000 the first year. The unit selling price to achieve this target profit must be at least A. $37.00 B. $36.60 C. $34.60 D. $39.00

Answer (D) is correct. If X represents the necessary selling price, 25,000 equals maximum sales volume, $16 is the variable cost per unit, $450,000 is the total fixed cost, and $125,000 [$75,000 target after-tax profit ÷ (1.0 - .4 tax rate)] is the desired pre-tax profit, the following formula may be solved to determine the requisite unit price: 25,000 (X - $16) - $450,000 = $125,000 25,000X - $400,000 - $450,000 = $125,000 25,000X = $975,000 X = $39

Total production costs of prior periods for a company are listed as follows. Assume that the same cost behavior patterns can be extended linearly over the range of 3,000 to 35,000 units and that the cost driver for each cost is the number of units produced. Cost X Cost Y 3,000 $23,700 47,280 Production in Units per Month 9,000 16,000 $ 52,680 141,840 $ 86,490 252,160 35,000 $178,260 551,600 The company is concerned about its current operating performance that is summarized as follows. Sales ($12.50 per unit) Variable costs Net operating loss $300,000 180,000 (40,000) How many additional units should have been sold in order for the company to break even? A. 32,000 B. 16,000 C. 12,800 D. 8,000

Answer (D) is correct. The breakeven point in units equals fixed costs divided by the difference between unit price and unit variable cost. Fixed costs were $160,000 [($300,000 sales - $180,000 VC) + $40,000 NOL], units sold equaled 24,000 ($300,000 sales ÷ $12.50 SP), and unit variable cost was $7.50 ($180,000 VC ÷ 24,000 units sold). Accordingly, the breakeven point in units was 32,000 [$160,000 FC ÷ ($12.50 SP - $7.50 unit VC)], and the additional units that should have been sold to break even equaled 8,000 (32,000 - 24,000).

Romashka, Inc. plans to introduce a new product. The marketing manager forecasts a unit selling price of $500. The variable cost per unit is estimated to be $100. In addition, there is a total of $110,000 fixed indirect manufacturing costs, and $150,000 in fixed operating costs associated with these units. What quantity will the company have to sell to break even? A. 220 units. B. 275 units. C. 520 units. D. 650 units

Answer (D) is correct. The breakeven point in units is determined as follows: Breakeven point = Fixed costs ÷ Unit contribution margin Romashka's breakeven quantity can therefore be derived thusly: Breakeven point = ($110,000 + $150,000) ÷ ($500 - $100) = $260,000 ÷ $400 = 650 units

Von Stutgatt International's breakeven point is 8,000 racing bicycles and 12,000 5-speed bicycles. If the selling price and variable costs are $570 and $200 for a racer, and $180 and $90 for a 5-speed respectively, what is the weighted-average contribution margin? A. $100 B. $145 C. $179 D. $202

Answer (D) is correct. The contribution margin is selling price minus variable costs.

(Refers to Fact Pattern #5) How many surge protectors (rounded to the nearest hundred) must Bruell Electronics sell at a selling price of $14 per unit to increase after-tax income by $30,000? Bruell Electronics' effective income tax rate is 40%. A. 10,700 units. B. 12,100 units. C. 20,000 units. D. 28,300 units.

Answer (D) is correct. The number of units to be sold to generate a specified pre-tax income equals the sum of total fixed costs and the targeted pre-tax income, divided by the unit contribution margin. Given a desired after-tax income of $30,000 and a tax rate of 40%, the targeted pre-tax income must be $50,000 [$30,000 ÷ (1.0 - .4)]. Unit variable costs total $8 ($3.25 + $4.00 + $.75), and UCM is $6 ($14 unit selling price - $8). Hence, the desired unit sales level is 28,333 [($120,000 + $50,000) ÷ $6]. Rounded to the nearest hundred, the answer is $28,300.

(Refers to Fact Pattern #5) How many surge protectors (rounded to the nearest hundred) must Bruell Electronics sell at a selling price of $14 per unit to gain $30,000 additional income before taxes? A. 10,700 units. B. 12,100 units. C. 20,000 units. D. 25,000 units.

Answer (D) is correct. The number of units to be sold to generate a specified pre-tax income equals the sum of total fixed costs and the targeted pre-tax income, divided by the unit contribution margin. Unit variable costs total $8 ($3.25 + $4.00 + $.75), and UCM is $6 ($14 unit selling price - $8). Thus, the desired unit sales level equals 25,000 units [($120,000 + $30,000) ÷ $6].


Kaugnay na mga set ng pag-aaral

Chapter 24: Management of Patients With Chronic Pulmonary Disease

View Set

Fundamentals Proctored Practice Part 1

View Set